Community Exxam 2 (Chp. 7-14)

Lakukan tugas rumah & ujian kamu dengan baik sekarang menggunakan Quizwiz!

Chapter 09

Evidence-Based Practice

An immigrant who takes metamizole (banned in the United States) for pain may experience life-threatening agranulocytosis. Which of the following actions would be taken by a nurse who employs cultural re-patterning? a. Complete a cultural assessment to identify any other dangerous medications that the client may be taking. b. Put this into perspective by considering that many drugs used in the United States cause agranulocytosis. c. Explain the harmful effects of metamizole and recommend an alternative medication for pain. d. Recognize that taking metamizole is common among persons living in Mexico and accept this as a cultural tradition.

Explain the harmful effects of metamizole and recommend an alternative medication for pain. Cultural re-patterning means that the nurse works with clients to help them reorder, change, or modify their cultural practices when the practice is harmful to them. Completing a cultural assessment involves learning more about the client's culture but does not address the need to consider changing or modifying cultural practices. In order to complete cultural re-patterning, the nurse has to take an action to resolve this potential problem.

Which of the following places best describes where the incidence of Vancomycin-resistant Staphylococcus aureus (VRSA) and methicillin-resistant S. aureus (MRSA) is currently rising? a. Areas where people share dressing or bathing facilities. b. Daycare centers and schools c. Long-term care facilities d. Senior citizen centers

A. Areas where people share dressing or bathing facilities. Vancomycin-resistant Staphylococcus aureus (VRSA) and methicillin-resistant S. aureus (MRSA) remain problems for people who acquire the bacteria in the hospital, but there is a growing incidence of community-acquired MRSA in places where people closely share facilities such as locker rooms, prisons, and other close bathing areas. Daycare centers, schools, long-term care facilities, and senior citizen centers are not places where the incidence of VRSA and MRSA is rising. These are not places where people are closely sharing dressing and bathing facilities.

For a bedridden Muslim patient, the nurse rearranges the room and moves the bed so that it faces toward Mecca for the patient's daily prayers. Which of the following is the nurse demonstrating through these actions? a. Accommodation b. Awareness c. Brokering d. Imposition

Accommodation Cultural accommodation involves including aspects of the patient's religious beliefs and/or folk practices in the traditional health care system to implement essential treatment plans. For this patient, daily prayer in the tradition of Islam is important—from the patient's perspective, possibly more important than medical treatment. Cultural awareness is the self-examination and in-depth exploration of one's own biases, stereotypes, and prejudices that influence behavior. Cultural brokering is advocating, mediating, negotiating, and intervening between the client's culture and the biomedical health care culture on behalf of clients. Cultural imposition is the belief in one's own superiority, or ethnocentrism, and is the act of imposing one's values on others.

At a local hospital, the postpartum care policy requires that nurses observe the mother during infant care to assess the mother's ability to care for the new baby and to promote bonding. A new mother expresses concern that in her country, all infant care is provided by other family members so that the mother can rest and recover. Which of the following actions would be taken by a culturally competent nurse? a. Allow family members to provide the newborn's care and assess the mother's knowledge of childcare through discussion. b. Reinforce the importance of bonding and that all good mothers gladly assume these responsibilities. c. Explain that the process of postpartum recovery does not require this much rest and require that she provide infant care. d. State that she must abide by hospital policy because documentation of the mother's ability to give the infant care is required for discharge.

Allow family members to provide the newborn's care and assess the mother's knowledge of childcare through discussion. Culturally competent nursing care focuses on the specific patient, reflects the patient's individual beliefs and values, and is provided with sensitivity. The nurse should accept the cultural norms and behaviors of this client and her family. Thus, the nurse should provide culturally competent care and comply with the client's desire. This should take priority over any judgments the nurse could make about this client or hospital documentation related to infant care by the mother. This respect of culture makes the other options incorrect.

A nurse is caring for a client of another culture. Which of the following actions would be the most appropriate for the nurse to take? a. Alter personal nonverbal behaviors to reflect the cultural norms of the client. b. Keep all behaviors culturally neutral to avoid misinterpretation. c. Rely on friendly gestures to communicate caring for the client. d. Avoid any pretense of prejudice by treating the client in the same way as any other client.

Alter personal nonverbal behaviors to reflect the cultural norms of the client. Cultural competence in nursing includes adoption of culturally congruent behaviors. Culturally skillful nurses use appropriate touch during conversation, modify the physical distance between themselves and others, and use strategies to avoid cultural misunderstandings while meeting mutually agreed-upon goals. Nurses who strive to be culturally competent are expected to respect other cultures and value diversity. These behaviors tend to provide more responsive care. Nurses should be knowledgeable of other cultures and communicate with the client based on cultural norms. Culturally skillful nurses understand the unique difference among individuals within a given culture and work with those individuals to learn more about their culture and provide culturally sensitive care.

Which of the following best describes most Americans' attitude toward immigrants? a. Ambivalence, because there are no clear solutions about how to address their needs. b. Strongly negative, because immigrants take jobs that native-born Americans could have instead. c. Strongly positive, because immigrants bring useful job skills and often join previous family members already in the United States. d. Strong opposition to further immigration, because of the increasing population in the United States.

Ambivalence, because there are no clear solutions about how to address their needs. Most Americans are ambivalent about immigration, recognizing both the positive and negative aspects involved and realizing that it is a complex issue that has no clear solutions. Because Americans are ambivalent, there is neither a strong negative nor a positive attitude toward immigrants. However, many times immigrants do enter the United States because they have useful job skills or family ties. They are more likely to be low-income workers who work in low-wage, blue-collar jobs and industries.

The nurse practitioner (NP) discovered that an immigrant client is not taking the penicillin prescribed because his illness is "hot" and he believes that penicillin, a "hot" medicine, will not provide balance. Which of the following terms best describes the action taken by the NP when the client's prescription is changed to a different yet equally effective antibiotic? a. Cultural awareness b. Cultural brokering c. Cultural knowledge d. Cultural skill

Cultural knowledge Cultural knowledge is information about organizational elements of diverse cultures and ethnic groups; emphasis is on learning about the client's worldview from an emic (native) perspective. Cultural skill involves the provision of care that is beneficial, safe, and satisfying to the client. The medication change allows the client to retain his cultural beliefs and also satisfies the nurse practitioner's need to prescribe an effective antibiotic. Cultural awareness is the self-examination and in-depth exploration of one's own biases, stereotypes, and prejudices that influence behavior. Cultural brokering is advocating, mediating, negotiating, and intervening between the client's culture and the biomedical health care culture on behalf of clients.

Chapter 07

Culture of Populations in Communities

Chapter 08

Environmental Health

Chapter 10

Epidemiological Applications

Chapter 11

Infectious Disease Prevention and Control

A nurse who is explaining to an immigrant client why it is important to take medication states, "The medication takes a couple of weeks to be effective, but then you should feel better." When the client is next seen, no medication has been purchased. Which of the following is the most likely explanation? a. The nurse emphasized that eventually the client would feel better, but the client needed to feel better immediately so didn't bother with the drug. b. The medication required a trip to the pharmacy, and the client just hadn't had time to obtain the drug yet. c. The medication was too expensive for the client's family. d. The client really hadn't understood why the medication was important.

The nurse emphasized that eventually the client would feel better, but the client needed to feel better immediately so didn't bother with the drug. If we look closely at what the nurse stated, there may have been a cultural disconnect based on time perception. Many nurses are future oriented, whereas many families may place greater value on quality of life and view present time as being more important. When nurses discuss health promotion and disease prevention strategies with persons from a present orientation, they should focus on the immediate benefits these clients would gain rather than emphasizing future outcomes. The cultural disconnect of time should be the immediate concern of the nurse. It is possible that the client did not have the necessary resource or did not understand the importance of the medication, but the nurse should first investigate the potential cultural disconnect.

Earlier in the week, a nurse carefully taught a patient from a different culture exactly how much medication to take and emphasized the importance of taking the correct amount. However, the patient is back in the hospital today with symptoms of an overdose although the patient denies taking more than the label indicated. Which of the following is the most likely explanation? a. The patient was taking more mediation in the hope of getting well faster. b. The patient was also taking folk medicines that had many of the same effects and perhaps some of the same ingredients as the prescribed medication. c. The patient truly did not understand and thought the dose being taken was correct. d. The patient had a unique response to the medication and should have a smaller dose ordered.

The patient was also taking folk medicines that had many of the same effects and perhaps some of the same ingredients as the prescribed medication. For fear of disapproval, a person may not tell the nurse that he or she is using folk medicine as well as Western medication. The two medicines may have cumulative effects that could be dangerous to the client. Nurses who lack cultural knowledge may develop feelings of inadequacy and helplessness because they are often unable to effectively help their clients. It is unlikely that the patient was taking too much medication, taking the incorrect dose, or having a unique reaction to the medication. Rather the nurse should first interview the patient about use of folk medicine which may interact with the prescribed medication regimen.

A nurse recognizes that although a patient speaks English, the patient is from a culture with which the nurse is unfamiliar. Therefore, a cultural assessment should be attempted. Which of the following questions should the nurse ask? (Select all that apply.) a. "Can you tell me where your family is from?" b. "Do you practice a particular religious faith?" c. "What other countries have you lived in?" d. "Is there anything special we need to know about your food preferences?" e. "What do you think is causing your health problem?"

a. "Can you tell me where your family is from?" b. "Do you practice a particular religious faith?" d. "Is there anything special we need to know about your food preferences?" e. "What do you think is causing your health problem?" In a general cultural assessment, nurses ask clients about their ethnic background, language, education, religious affiliation, dietary practices, family relationships, hospital experiences, occupation and socioeconomic status, cultural beliefs, and language. Nurses want to also ask about the client's perception of the health issue and what caused it and how it should be treated as well as the results they expect from the care they get. Such basic data help nurses understand the client from the client's point of view and recognize what is unique about the person, thus avoiding stereotyping. Knowing the other countries that the client has lived in may be helpful; however, necessary information about the client's culture should be able to be collected through broad questions about the client's culture.

Statistics clearly demonstrate that there are significantly more cases of a disease in one particular neighborhood than in all the rest of the city. Assuming all else is the same, which of the following is the most likely explanation for a single neighborhood having such a different pattern of illness? a. A cultural or ethnic concentration in the neighborhood b. The geographic location of the neighborhood within the city c. A statistical fluke without meaning. d. The time of year the different statistics were collected throughout the city.

a. A cultural or ethnic concentration in the neighborhood The most probable reason is that there is a cultural or ethnic concentration in that particular neighborhood that has a different lifestyle pattern, resulting in different health outcomes. The assumption is made that all things in the city are the same, thus the geographical location would not be a likely explanation for the difference. However, location may play a role at times with increased incidence of disease depending on exposure to certain environmental factors which could place the population at risk. A statistical fluke does not provide a likely explanation for the difference. There are times when there are cyclical patterns of disease. However, the question states that the assumption should be made that all else is the same, so one can assume that the data were collected within the city at the same time.

A nurse believes a new mouth care procedure (MCP) is exacerbating a client oral tissue problems. Which of the following must be present for the nurse to go to administration with confidence that the MCP is causing problems? (Select all that apply.) a. A plausible explanation of how the new MCP could cause harm. b. A strong feeling that the MCP is the cause. c. Consistently seeing mouth inflammation in many of the patients who have received the MCP. d. Documentation from patient records that mouth inflammation in clients did not occur until after the new procedure was implemented. e. A less expensive option for appropriate oral care

a. A plausible explanation of how the new MCP could cause harm. c. Consistently seeing mouth inflammation in many of the patients who have received the MCP. d. Documentation from patient records that mouth inflammation in clients did not occur until after the new procedure was implemented. Strength of association is suggested by the fact that patients who did not receive the MCP and patients seen on the floor before the new MCP did not have problems, whereas patients who received the new MCP are having problems. Seeing the problems in many of the patients suggests a consistency. The fact that those who had the procedure more often have worse problems suggests a dose-response relationship. A plausible explanation of how the new procedure could cause harm enhances the biological risk. Although one would hope that previous testing would have been done before the product was released to market, the product could be safe for healthy persons but a risky process for those with compromised immunity or those who are under stress. Feelings alone are not convincing. A study should be set up to confirm or dispute the nurse's hypothesis. Cost of the initial procedure is not a relevant factor.

Which of the following strategies would a nurse expect to be implemented by a typical state environmental agency? (Select all that apply.) a. Acting, through unannounced inspections, to ensure compliance. b. Completing a community assessment. c. Monitoring hazardous substances to uphold established standards. d. Obtaining and analyzing samples to confirm compliance e. Involvement in the permitting process established by the state.

a. Acting, through unannounced inspections, to ensure compliance. c. Monitoring hazardous substances to uphold established standards. d. Obtaining and analyzing samples to confirm compliance. e. Involvement in the permitting process established by the state. The organization and approach to environmental protection vary somewhat among states, but the common essential strategies of prevention and control via the permitting process, establishment of environmental standards, and monitoring, as well as compliance and enforcement, are found in every state. Completing a community assessment is not a strategy that is typically implemented by a state environmental agency, rather that typically occurs at the local level.

Which of the following explains why contagious infections are becoming a central focus of public health? (Select all that apply.) a. Americans are fearful of terrorists using biological agents. b. Awareness of human susceptibility to animal diseases has been publicized. c. Drug-resistant strains of old diseases have evolved. d. Media coverage exaggerates the dangers of exposure to crowds. e. The American population has become more health conscious in the last 20 years.

a. Americans are fearful of terrorists using biological agents. c. Drug-resistant strains of old diseases have evolved. New infectious diseases and new forms of old diseases, such as drug-resistant strains of TB, have emphasized the dangers of infectious diseases. Potential threats from terrorist use of infectious agents have also emphasized infectious diseases. There has not been any increased awareness of human susceptibility to animal diseases that has caused contagious infections to become a central focus of public health. Also, media coverage has not exaggerated the danger of exposure to crowds when it comes to contagious infections. While health and wellness is a higher priority for many Americans, the interest is not a factor in focus on contagious infections.

Which of the following actions would a nurse take when serving as an advocate for the community? (Select all that apply.) a. Asking questions related to health implications at policy meetings. b. Calling the local health department to report problems at the hospital. c. Serving as a source of information at public meetings d. Volunteering to serve on health-related committees. e. Conducting a health-related screening at a day care center.

a. Asking questions related to health implications at policy meetings. c. Serving as a source of information at public meetings d. Volunteering to serve on health-related committees. Advocacy roles of the community-oriented nurse include attending policy meetings to obtain health-related information, holding public meetings (or serving on panels at meetings) to provide health-related information, serving on health-related committees, and informing local media about environmental hazards in the community. Calling the local health department to report problems at the hospital and conducting a screening at a local day care center do not demonstrate the nurse advocating for the community as the hospital and the day care do not represent the entire community.

Staff members have agreed to implement evidence-based practice; they have chosen a specific problem and searched the literature. The group has selected the interventions that seem the easiest to implement. Which of the following actions would the staff take next? a. Assess the quality of the evidence in the literature. b. Decide how best to orient the staff and community to the proposed changes. c. Eliminate all nursing interventions that are not evidence based. d. Choose another specific problem for the next literature search.

a. Assess the quality of the evidence in the literature. After the group has chosen the topic and evaluated the literature for approaches that seem feasible, specific interventions are chosen. The quality of the evidence must be assessed before recommending specific changes or writing a protocol to resolve the problem. Grading the strength of evidence or determining the quality, quantity, and consistency of research studies must be done before making recommendations for practice. Deciding how to orient staff and the community to the change would occur at the end of the process after the findings have been summarized, and written recommendations or protocols have been developed. Elimination of nursing interventions that are not evidence based would be part of the written recommendations and proposal that happen at the end of the process. The nurse should not plan another literature search or topic to explore until after completing the process with the current issue.

A nurse is familiar with evidence-based practice (EBP) and wants to implement it into the care of clients. Which of the following would present the biggest challenge? a. Assessing one's current practice and accessing evidence-based resources. b. Convincing administration that EBP is beneficial. c. Distinguishing EBP from practice based on old standards. d. Showing clients that EBP will improve their health outcomes.

a. Assessing one's current practice and accessing evidence-based resources. The first step toward implementing evidence-based practice in nursing is recognizing the current status of one's own practice and believing that care based on the best evidence will lead to improved client outcomes. After the nurse has assessed his or her own practice and accessed EBP resources, the next steps would be to talk with administration and implement EBP into practice. Without self-reflection first, the nurse would be ineffective in implementing EBP. The challenge for the clinician is how to access the evidence and integrate it into practice, thus moving beyond practice based solely on experience, tradition, or ritual.

Which of the following are the major sources of air pollution in the United States? (Select all that apply.) a. Burning of fossil fuels. b. Waste incineration c. Wood burning fireplaces. d. Motor vehicles e. Coal-fired power plants.

a. Burning of fossil fuels. b. Waste incineration d. Motor vehicles e. Coal-fired power plants. Motor vehicles are the greatest single source of air pollution in the United States. The burning of fossil fuels (diesel, industrial boilers, and power plants) and waste incineration are two other major contributors. Wood burning fireplaces are not one of the major sources of air pollution that has been identified.

A community citizen reports to the public health nurse that the city water in one neighborhood has had an unusual taste for the past few months. Which of the following actions should the nurse take first? a. Check the most recent consumer confidence report. b. Consult the Centers for Disease Control and Prevention. c. Notify the Environmental Protection Agency (EPA). d. Place a call to the poison control center.

a. Check the most recent consumer confidence report. The consumer confidence report (also known as the right-to-know report) reports the condition of drinking water. Because this is only one citizen, whose sense of taste may be affected by many conditions, it would not be appropriate to overreact by calling in governmental agencies or poison control first. Even so, in case one needed to contact an authority, it would be appropriate to start with local governmental agencies such as the city water department rather than federal agencies.

A nurse implements an education program that incorporates computer games to reinforce learning for a group of older adults. Although the research demonstrates evidence of improved retention of this information, the nurse experiences exactly the opposite with this group. Which of the following is the most likely cause of such poor outcomes? a. Failure to consider client and setting differences. b. Inadequate incorporation of evidence into practice c. Inferior quality of the available research evidence d. Lack of skills when evaluating the evidence

a. Failure to consider client and setting differences. EBP cannot be applied as a universal remedy without attention to client differences. When EBP is applied at the community level, the best evidence may point to a solution that is not sensitive to cultural issues and distinctions and thus may not be acceptable to the community. For example, computer games may be excellent for younger groups but are often poorly suited for older Americans who may face challenges with learning new technology. Because the nurse did not consider the client differences, the intervention was unsuccessful. This does not mean that the research had poor quality, was not evaluated correctly, or was not correctly incorporated into practice. The nurse should not assume that an EBP intervention will work in a different setting or with a different population than what it has been intended.

A nurse has decided to increase the evidence base of current nursing practice in an agency. Which of the following describes a barrier that could be encountered by the nurse? (Select all that apply.) a. Colleagues who do not know how to search the literature or critique research. b. Dedication to the history and tradition of the agency c. Little or no research published in the clinical area of concern. d. Several meta-analyses in the literature with inconsistent results e. The agency is unable to provide sufficient funds to support the plan.

a. Colleagues who do not know how to search the literature or critique research. b. Dedication to the history and tradition of the agency c. Little or no research published in the clinical area of concern. Barriers to evidence-based practice exist when the following are limited or lacking: time, access to journal articles, search skills, critical appraisal skills, and an understanding of research terminology. Other barriers include miscommunication about the process; inferior or unavailable research or other evidence; unwillingness of organizations to fund research or make decisions based on evidence; and concern that evidence-based practice will decrease emphasis on individual client needs or the nurse's clinical decisions. Dedication to the history and tradition of the agency may pose a barrier as this may influence the philosophy of the practice environment and the willingness to embrace EBP. The numbers of meta-analyses in the literature, regardless if the results are consistent or not, would not be a barrier to the practice environment when implementation is considered.

Which of the following actions represents the use of secondary prevention to reduce environmental health risks? a. Collecting blood specimens from preschool children to check for lead levels. b. Meeting with local government officials to request that the city clean up a hazardous vacant lot. c. Referring a child with toxic lead levels to a neurologist. d. Teaching parents of a 2-year-old about the dangers of lead-based paint in older homes.

a. Collecting blood specimens from preschool children to check for lead levels. Secondary prevention refers to actions such as surveillance and screening, which are undertaken so that problems may be detected at early stages. Meeting with local government officials to request that the city clean up a hazardous vacant lot and referring child with toxic lead levels to a neurologist are examples of tertiary prevention as the problem already exists. Education to avoid exposure is part of primary prevention which relates to teaching parents about the dangers of lead-based paint in older homes.

A city has announced its plans to build a city dump near a community of poor and predominantly African-American citizens. Which of the following principles would the nurse be using when vocalizing opposition for this plan? a. Environmental justice b. Environmental epidemiology c. Tertiary prevention d. Risk communication

a. Environmental justice Environmental justice is the goal of campaigns seeking to improve the unequal burden of environmental risks borne by impoverished and minority communities. The Environmental Justice Act would be used in support of the nurse's position. Environmental epidemiology is concerned with the discovery of environmental exposures that contribute or protect against disease or illness which is not accomplished by the nurse vocalizing opposition to the plan. Vocalizing opposition to the plan would not be a tertiary prevention strategy as the nurse is addressing a problem that does not yet exist. Risk communication includes general principles of good communication (right information, to the right people, at the right time). In this scenario, the nurse is vocalizing opposition, not communicating risks.

A nurse is examining all of the various factors which can lead to disease. Which of the following models would the nurse most likely use? a. Epidemiologic triangle b. Health promotion c. Levels of prevention d. Natural history of disease

a. Epidemiologic triangle The epidemiologic triangle categorizes factors as agent, host, or environment. The model encourages the health care provider to examine all the influences that lead to increased risk. Levels of prevention are actions taken to improve health outcomes. Health promotion addresses health improvement, not the risk for disease.

An American nurse says, "I'm not going to change the way I practice nursing based on where the client is from because research shows that Western health care technology and research is best." Which of the following is being demonstrated by the nurse's statement? a. Ethnocentrism b. Prejudice c. Racism d. Stereotyping

a. Ethnocentrism Ethnocentrism, a type of cultural prejudice at the cultural population level, is the belief that one's own group determines the standards for behavior by which all other groups are to be judged. For example, some American nurses and providers may think, "The way we do it is the only right way to provide this care." Prejudice refers to having a deeply held reaction, often negative, about another group or person. Racism refers to the belief that persons who are born into a particular group are inferior in intelligence, morals, beauty, or self-worth. Stereotyping occurs when attributing certain beliefs and behaviors about a group to an individual without giving adequate attention to individual differences.

The administration at a local medical center examines the trends in health problems when developing long-range plans for staffing and space allocation. Which of the following sources of information would be most helpful? a. Local data drawn from a professional survey in the city. b. The National Health Interview Survey c. The National Hospital Discharge Survey d. The state's vital statistics

a. Local data drawn from a professional survey in the city. For many studies, however, the only way to obtain the needed information is to collect the required data in a study specifically designed to investigate a particular question. Both the National Health Interview Survey and the National Hospital Discharge Survey provide information on the health status and behaviors of the national population. A state's vital statistics are the birth and death certificates within the state which would not give the administration the information that they are looking for in this community.

When teaching a nutrition class to a student group with a large Latino population, the school nurse incorporates foods such as salsa and other healthy dishes familiar to students into the presentation. Which of the following best describes the action taken by the nurse? a. Primary prevention b. Secondary prevention c. Tertiary prevention d. Both primary and secondary preventions

a. Primary prevention Primary prevention involves activities such as health teaching to prevent a problem from occurring. Secondary prevention occurs when a nurse completes screenings. Tertiary prevention occurs after a problem has occurred and aims to restore the client to the highest level of functioning as possible.

Which of the following is the best way to increase the number of persons who come to their screening test appointments? a. Reminding clients via telephone, e-mail, or mail b. Emphasizing long life and happy family when conditions are caught early and treated successfully. c. Pointing out how inexpensive and convenient screening tests are d. Stressing the dangerousness of the condition if not caught early

a. Reminding clients via telephone, e-mail, or mail Client reminders and recalls via telephone, e-mail, or mail—or a combination of these strategies—are effective in increasing compliance with screening activities such as those for colorectal and breast cancer. Emphasizing the importance of screening, how it is inexpensive, and the dangers of the condition if it is not diagnosed early can all occur as clients are reminded of the screening via telephone, e-mail, or mail. Without the reminder, clients are likely to lose sight of the benefits of receiving the screening.

Two women seem to agree on almost everything from favorite music to favorite media stars to the best way to prepare a meal. Which of the following best explains this similarity in the two women? a. They are both members of the same birth cohort. b. They are close friends. c. They attended the same school. d. They both go to the same church.

a. They are both members of the same birth cohort. Being born at about the same time would mean both women have lived through similar social events and media occurrences and therefore would have much in common. Being close friends is probably the result of the similarity rather than the cause. Going to the same school or the same church, depending on the size of the institutions, might not result in any personal interaction whatsoever.

Which of the following is the number one cause of death worldwide? a. Chronic diseases (heart disease, cancer, stroke) b. Infectious diseases c. Injuries (accidental or purposeful) d. Terrorism

b. Infectious diseases Infectious diseases, however, are still the number one cause of death worldwide. In countries with higher standards of living, where people live longer, chronic diseases—heart disease, cancer, and stroke—are the leading causes of death. Injuries and terrorism are not the number one cause of death worldwide.

An undocumented immigrant comes to a primary care provider's office to receive care. Which of the following services can the client receive? (Select all that apply.) a. Treatment for tuberculosis b. Treatment for type 2 diabetes c. Immunization for polio d. Physical examination e. Sutures for a laceration

a. Treatment for tuberculosis c. Immunization for polio e. Sutures for a laceration Undocumented immigrants or illegal aliens are individuals who have crossed a border into the United States illegally or whose legal permission to stay in the United States has expired. They are eligible only for emergency medical services, immunizations, treatment for the symptoms of communicable diseases, and access to school lunches. Undocumented immigrants are not eligible to receive treatment for chronic diseases such as type 2 diabetes or physical examinations.

A nurse states, "The best way to treat a client from another country is to care for them the same way we would want to be cared for. After all, we are all humans with the same wants and needs." What does this statement reflect in relation to culture? a. Awareness b. Blindness c. Knowledge d. Preservation

b. Blindness Cultural blindness is the tendency to ignore differences between cultures and to act as if they do not exist. People from different cultures may have different expectations, wants, and needs. Cultural awareness is the self-examination and in-depth exploration of one's own biases, stereotypes, and prejudices that influence behavior. Cultural knowledge is information about organizational elements of diverse cultures and ethnic groups; emphasis is on learning about the client's worldview from an emic (native) perspective. Cultural preservation means that the nurse supports and facilitates the use of scientifically supported cultural practices from a person's culture along with those from the biomedical health care system.

A man is diagnosed with prostate cancer. Which of the following data should the nurse know to answer the man when he asks, "What are the chances I'll survive this thing?" a. Attack rate b. Case fatality rate c. Cause-specific morbidity rate d. Crude mortality rate

b. Case fatality rate The case fatality rate (CFR) is the proportion of persons diagnosed with a particular disorder (i.e., cases) who die within a specified period. The CFR is considered an estimate of the risk for death within that period for a person newly diagnosed with the disease. Persons diagnosed with a particular disease often want to know the probability of surviving. The CFR provides that information. The attack rate is the proportion of persons exposed to an agent who develop the disease. The cause-specific mortality rate is an estimate of the risk for death from some specific disease in a population. The crude mortality rate is an estimate of the risk for death for a person in a given population.

Which of the following statements about children and environmental hazards is correct? a. The prevalence of autism is directly related to the increase in environmental toxins. b. Children are more susceptible to environmental toxins because of their smaller size. c. The incidence of asthma among children has been decreasing. d. Children are more susceptible to cancer if they have a family history of the disease.

b. Children are more susceptible to environmental toxins because of their smaller size. Because of the smaller size of children, they are exposed to higher doses of pesticide residues in the foods they eat and drink. Infants and young children drink more fluids per body weight than adults, and this increases the dose of contaminants in their drinking water, milk (hormones and antibiotics), and juices (particularly pesticides). Only a small percentage of childhood cancers are associated with heredity. However, exposure to ionizing radiation increases the risk of childhood leukemia and possibly other cancers. All of the causes of autism spectrum disorder are not currently known. Environmental factors are thought to be a possible cause, as are biologic and genetic factors. Asthma is common among children, and the strongest risk factors are genetic factors and inhaled substances and particles that provoke an allergic reaction or irritate the airways. Indoor air quality is a growing concern because of the alarming rise in the incidence of asthma in the United States, particularly among children.

Which of the following statistics is used by countries to compare the success of their health care systems? a. Attack rate b. Infant mortality rate c. Proportionate mortality ratio d. Cause-specific mortality rate

b. Infant mortality rate Infant mortality is used around the world as an indicator of overall health and availability of health care services. The attack rate is the proportion of persons exposed to an agent who develop the disease. The cause-specific mortality rate is an estimate of the risk for death from some specific diseases in a population. The proportionate mortality ratio is the proportion of all deaths resulting from a specific cause.

A nurse wants to use the principles of risk and outrage to improve an environmental hazard in the community. To accomplish this, which of the following actions would be taken by the nurse? (Select all that apply.) a. Advertise in the media throughout the entire surrounding area. b. Communicate the correct information in a timely fashion. c. Share all the data found on the community assessment. d. Talk to those affected or those worried about the situation. e. Conduct a community-based survey to identify risks.

b. Communicate the correct information in a timely fashion. d. Talk to those affected or those worried about the situation. The correct information must be given in a language the audience—namely those at risk or worried about the risk—can understand. Use the communication channels the neighborhood residents use and meet at a common meeting place of the community. A survive is appropriate, but the question is focused on distribution of the found information. It is wasteful of resources to advertise in media throughout a wider region or to meet at a central regional facility when only community residents will be interested or involved. Sharing all the data would be overwhelming, and much of the data might not be relevant. Using epidemiological statistics would not be meaningful to those without the education or experience to be able to draw an appropriate conclusion.

A school health nurse plans to use evidence-based practice (EBP) to guide the development of health education programs most likely to increase retention of learning in elementary schoolchildren. Which of the following would be the best way to use EBP in this situation? a. Ask other school health nurses what they included in their own education programs. b. Compare and contrast randomized controlled trials related to learning in elementary schoolchildren. c. Develop a series of games to accompany the programs developed to promote health. d. Seek out and examine health education programs for elementary schoolchildren on the Internet.

b. Compare and contrast randomized controlled trials related to learning in elementary schoolchildren. Randomized controlled trials are generally ranked as the highest level of evidence. EBP is not collected by word of mouth; rather the nurse must look in the literature to obtain the best information. There is no evidence that a series of games will assist with the retention of learning based on what is stated in this question. This may be an appropriate activity, but this information must be gathered from the literature. Searching the Internet for ideas is helpful only if evidence-based practice sites are accessed, and most Internet sites are not EBP sites.

A nurse wishes to develop cultural competence. Which of the following actions should the nurse take first? a. Complete a survey of all the various ethnicities represented in the nurse's community. b. Consider how the nurse's own personal beliefs and decisions are reflective of his or her culture. c. Invite a family from another culture to join the nurse for an event. d. Study the beliefs and traditions of persons living in other cultures.

b. Consider how the nurse's own personal beliefs and decisions are reflective of his or her culture. Cultural awareness requires self-examination and an in-depth exploration of one's own beliefs and values as they influence behavior. Cultural awareness is the first element in the model of cultural competence. Following the development of cultural awareness, the next step is cultural knowledge in which information about organizational elements of diverse cultures and ethnic groups is collected. The next stage of the model, cultural skill, occurs with the effective integration of cultural awareness and cultural knowledge to obtain relevant cultural data and meet the needs of culturally diverse clients. The fourth construct essential to this model is cultural encounter, which is the process that permits nurses to seek opportunities to engage in cross-cultural interactions with clients of diverse cultures to modify existing beliefs about a specific cultural group.

A health care provider is concerned about the high number of clients with type 2 diabetes mellitus who have poor glucose control. What would be the best reference for the provider to implement evidence-based practice (EBP) in the management of this problem? a. Published protocols. b. Current research findings c. Opinions of colleagues d. Nursing journals

b. Current research findings EBP in community-oriented nursing challenges nurses to integrate outcomes of the best evidence into their clinical practice. Current research findings will explicate evidence of the most successful interventions. (Randomized controlled trials are the gold standard of research for EBP). Protocols and opinions often reflect tradition rather than the most current scientific evidence. Although many nursing journals are peer reviewed, many are not and, even if peer reviewed, many are not research based or focused on scientific evidence.

A nurse is using analytic epidemiology when conducting a research project. Which of the following projects is the nurse most likely completing? a. Reviewing communicable disease statistics. b. Determining factors contributing to childhood obesity. c. Analyzing locations where family violence is increasing. d. Documenting population characteristics for healthy older citizens.

b. Determining factors contributing to childhood obesity. Epidemiology refers not only to infectious epidemics but also to other health-related events. The goal of analytic epidemiology is to discover the determinants of outcomes—the how and the why. Analytic epidemiology looks at the etiology (origins or causes) of disease. It discusses the disease in terms of how and why. Descriptive epidemiology considers health outcomes in terms of what, who, where, and when. It discusses a disease in terms of person, place, and time.

A school nurse is developing a primary prevention strategy for school-aged children. Which of the following interventions would the nurse most likely implement? a. Developing individualized exercise programs for overweight children b. Drafting policy for increases in noncompetitive physical activity programs. c. Monitoring body mass index (BMI) in children to identify elevations before they become difficult to manage. d. Notifying parents and/or guardians of their child's height-weight scale in comparison with national norms

b. Drafting policy for increases in noncompetitive physical activity programs. At the primary prevention level, campaigns to support regular exercise, greater emphasis on school-based physical education programs, and environmental policy initiatives to create or enhance places for physical activity in communities can make significant contributions to improving the lifestyle of sedentary children. Exercise programs are an example of tertiary prevention. Monitoring BMI in children is an example of secondary prevention. Notifying parents of their children's height-weight scale increases family awareness but does not meet the definition of a preventive measure.

A busy school health nurse concerned over the rising incidence of obesity wants to implement evidence-based practice (EBP) but faces barriers because of time constraints. Which of the following actions should the nurse take first? a. Identify students who are obese so that they may be closely monitored for weight control success or failure. b. Evaluate best practices to determine those that have the highest success rates for weight control in children. c. Schedule physician appointments for obese children. d. Develop an obesity management program for children whose body mass index exceeds normal.

b. Evaluate best practices to determine those that have the highest success rates for weight control in children. The first step of the seven-step EBP process is step zero, which involves a curiosity about the interventions that are being applied; this is not described in any of the topic descriptors. Step one requires asking questions in a "PICOT" format; this is not described in any of the topic descriptors. Step two involves searching for the best evidence to answer the question. This is done through evaluating best practices. The first step of the seven-step EBP process is step zero, which involves a curiosity about the interventions that are being applied; this is not described in any of the topic descriptors. Step one requires asking questions in a "PICOT" format; this is not described in any of the topic descriptors. Step two involves searching for the best evidence to answer the question. This is done through evaluating best practices.

A nurse is employed as a nurse epidemiologist. Which of the following activities would most likely be completed by the nurse? a. Eliciting the health history of a client presenting with an illness b. Evaluating the number of clients presenting with similar diseases. c. Performing a physical examination of an ill client. d. Providing treatment and health education to a client with a disease.

b. Evaluating the number of clients presenting with similar diseases. Epidemiology monitors the health of the population by examining measures of morbidity, especially incidence proportions, incidence rates, and prevalence proportions and learning about the risk for disease, the rate of disease development, and the levels of existing disease in a population, respectively. Epidemiology differs from clinical medicine, which focuses on the diagnosis and treatment of disease in individuals.

A nurse is planning to host a health screening at a large urban mall. Which of the following variables will help the nurse determine which screenings should be included? (Select all that apply.) a. Adequate space for persons to lie down after testing until side effects are reduced. b. Health problems for which the specific population is at risk c. Whether adequate privacy can be obtained for the invasive or embarrassing procedures d. Whether health care providers are available to follow up on any positive screening results e. Ability to provide the individual health screenings free to the public.

b. Health problems for which the specific population is at risk. d. Whether health care providers are available to follow up on any positive screening results. The screening tests should be reliable, valid, fast, and inexpensive. Results should be known immediately. Ethically, nurses should not screen for any problem unless they can refer those with positive results to a source for follow-up testing and treatment. The screening tests should have few side effects, be minimally invasive, and be capable of detecting enough new cases to warrant the effort and expense. No tests should be used that have negative side effects, are invasive, or cause embarrassment.

Which of the following groups is pressuring clinicians the most to use evidence-based practice (EBP)? a. Administrators b. Insurance companies c. Nurses d. Physicians

b. Insurance companies Much of the pressure to use evidence-based practice comes from third-party payers and is a response to the need to contain costs and reduce legal liability. Nurses, physicians, and administrators are not pressuring the use of EBP as much as insurance companies as these are not the payers of health care.

After finding several studies related to the clinical problem, a nurse knows the studies must be evaluated. Which of the following characteristics should be present in the literature? a. Federally supported multiagency clinical studies. b. Multiple high-quality studies with large sample sizes and consistent findings c. Research studies done by multidisciplinary teams in multiple settings. d. Controlled clinical trials.

b. Multiple high-quality studies with large sample sizes and consistent findings The Agency for Healthcare Research and Quality (AHRQ) reviewed 40 systems used to evaluate the quality of studies and strength of evidence. The report identified three domains for evaluating systems that grade the strength of evidence: quality, quantity, and consistency. The quality of a study refers to the extent to which bias is minimized. Quantity refers to the number of studies, the magnitude of the effect, and the sample size. Consistency refers to studies that have similar findings, using similar and different study designs. Federally supported multiagency clinical studies, research studies completed by multidisciplinary teams in multiple settings, and controlled clinical trials do not necessarily meet the three domains for evaluating systems to grade the strength of the evidence: quality, quantity, and consistency. These are the criteria that should be used to evaluate the literature, so additional information is needed about these studies in order to evaluate them.

A nurse wants to find information about environmental threats that are present in the community. Which of the following would be the best source of data for the nurse? a. Cumulative Index of Nursing and Allied Health (CINAHL) database b. National Library of Medicine c. State health department d. Closest local library

b. National Library of Medicine Technology helps us understand environmental threats. The National Library of Medicine (NLM) databases are user-friendly and accessible on the Internet. The NLM website provides access to medical databases such as PubMed and GratefulMed, which can be searched for possible environmental linkages to illnesses using key terms. The CINAHL database provides access to literature related to nursing and allied health journals and would not be as comprehensive as the National Library of Medicine. It would be difficult to access information easily through the health department. The information that is available at the local library would be dependent on the community in which one lived.

A nurse is assessing potential environmental health risks in the community. Which of the following would be the first step that the nurse should take? a. Conduct health risk assessments of randomly selected individuals. b. Perform a windshield survey. c. Review facility permits and consumer confidence reports. d. Survey community members.

b. Perform a windshield survey. Conducting a windshield survey is a useful first step to understanding potential environmental health risks. This provides firsthand information about the community and areas of concern that must be investigated. Conducting health risk assessments of randomly selected individuals or surveying community members does not give the nurse a good assessment of potential environmental health risks of the entire community. Reviewing facility permits and consumer confidence reports would not provide enough information to assess the environment of a community.

A community health nurse is determining the best way to address an outbreak of a new infectious disease using evidence-based practice (EBP). Which of the following actions would the nurse most likely take? a. Review policies and procedures. b. Review outcomes of clinical trials. c. Review several nursing textbooks. d. Review reputable sites on the Internet.

b. Review outcomes of clinical trials. Research findings, knowledge from basic science, clinical knowledge, and expert opinion should be considered sources of evidence for EBP. The use of policies and procedures is only helpful if they have been written using EBP, and depending on when they were written, there may be evidence that supports the use of a different practice. The problem with nursing textbooks is that many are not grounded in evidence-based practice, because the concept is relatively new to the United States. Scanning the Internet for ideas is helpful only if evidence-based practice sites are accessed, and most internet sites are not EBP sites.

A nurse is completing a basic health assessment. Which of the following questions should be asked by an environmentally aware nurse? a. "Is anyone else in your family having these symptoms?" b. "How many people live in your home?" c. "What jobs have you held the longest?" d. "Where did you live 10 years ago?"

c. "What jobs have you held the longest?" An exposure history should identify current and past exposures, have a preliminary goal of reducing or eliminating current exposures, and have a long-term goal of reducing adverse health effects. The "I PREPARE" mnemonic consigns the important questions to categories that can be easily remembered. Asking about previous employment helps to answer the "P" of past work. The other questions do not address the topics identified in the "I PREPARE" mnemonic (investigate potential exposures, present work, residence, environmental concerns, past work, activities, referrals, and resources, and educate).

A nurse is concerned about the high incidence of sexually transmitted infection (STIs) in the community college population and sets up a special STI screening. Which of the following groups of students would be encouraged to attend? (Select all that apply.) a. Sexually active students currently receiving treatment for an STI. b. Sexually active students who had been screened the previous year. c. Students who are not sexually active and do not plan to become sexually active. d. Students who are sexually active but never "go all the way." e. Students who are freshmen or new transfers.

b. Sexually active students who had been screened the previous year. d. Students who are sexually active but never "go all the way." Those who are at risk to develop the problem should be encouraged to attend the screening. This should be anyone who is or plans to be sexually active to any degree. Those already diagnosed with the problem are not at risk, because they already have the condition and are no longer at risk for developing it. Thus, it would not be beneficial for this population to attend the screening as they have already been diagnosed. Students who are not sexually active do not need to attend the screening as they are not at risk for contracting an STI because they are not having sex.

Which of the following actions by Florence Nightingale demonstrates her role as an epidemiologist? a. She convinced other women to join her in giving nursing care to all the soldiers. b. She demonstrated that a safer environment resulted in decreased mortality rate. c. She obtained safe water and better food supplies and fought the lice and rats. d. She met with each soldier each evening to say goodnight, thereby giving psychological support.

b. She demonstrated that a safer environment resulted in decreased mortality rate. Nightingale examined the relationship between the environment and the recovery of the soldiers. Using simple epidemiological measures, she was able to show that improving environmental conditions and adding nursing care decreased the mortality rates of the soldiers. Nightingale used statistics to document decreased mortality rates when the environmental factors were improved. Asking other women to assist with providing nursing care, obtaining safe water, and meeting with each soldier are not interventions that demonstrate the use of epidemiology (the relationship between the health events and the determinants or factors that influence those patterns)

In a particular community, several high school students were diagnosed with diabetes mellitus type 2 during the annual high school health fair. Over the next few years, the nursing staff developed and implemented educational programs about the risk factors for diabetes mellitus type 2 and proper nutrition. Which of the following would be the most useful for the nurses to use to determine if they are having any impact? a. The epidemic of diabetes in the high school is gradually ending. b. The incidence of diabetes is slowly decreasing during screening events. c. The prevalence of diabetes is slowly decreasing during screening events. d. The risk for diabetes is slowly increasing over time.

b. The incidence of diabetes is slowly decreasing during screening events. Incidence rates and incidence proportions are the measure of choice to study etiology because incidence is affected only by factors related to the risk of developing disease and not to survival or cure. If the educational programs are having the desired impact, the incidence of diabetes being diagnosed will decrease in future screenings. Prevalence is a fairly stable number over time, but incidence reacts more quickly to changes in risk factors or intervention programs. In order for an epidemic to have occurred, the rate of the illness would have had to exceed the usual level of that condition. There is no evidence in this question to support that the disease was at an epidemic level in this population. If the educational programs are effective, the risk for diabetes should be slowly decreasing over time.

The nursing staff has attempted to screen the entire African-American population in the community for diabetes. Which of the following would provide immediate verification of the success of the nursing staff's efforts? a. An epidemic of diabetes will be recognized. b. The incidence of diabetes will increase in the community. c. The prevalence of diabetes will decrease in the community. d. The risk for diabetes in the community will increase.

b. The incidence of diabetes will increase in the community. If the screening has been successful, more diabetes will be diagnosed and, hopefully, treated. Thus, the incidence of new cases will increase. Overall, prevalence will also increase, but that is not one of the answer options. An epidemic occurs when the rate of disease, injury, or other conditions exceeds the usual level of that condition. The prevalence (measure of existing disease in a population at a particular time) of diabetes would also increase. The risk for diabetes would not increase rather it would be more likely that the disease would be detected.

A patient who identifies as Buddhist enters the hospital for diagnostic testing just before lunch time. The nurse tells the aide to give a meal tray to the new patient, because no tests will be done until later that evening. The aide gives the patient a meal of Salisbury steak, bread, green beans, and potatoes with brown gravy. The patient eats nothing but a slice of bread and the green beans. Which of the following considerations was omitted by the nurse? a. The patient should not be served any food until a primary care provider's order is obtained. b. The patient's Buddhist faith probably requires a vegetarian diet. c. The patient may be too frightened about the tests to want to eat very much. d. The patient may have diabetes or be allergic to some foods

b. The patient's Buddhist faith probably requires a vegetarian diet. Although it is always wise to check with a patient before sending in food, the meal given to this patient was offensive. Most Buddhists are vegetarians and don't eat meat. The nurse should be aware of the cultural considerations that should be made for Buddhist patients. The nurse should ask the client about dietary restrictions before ordering a meal for the client so that these considerations can be made. The nature of the test determines the fasting requirements; no order is needed. Whether the client is diabetic or allergic to some foods should be determined upon admission

A public health nurse found that out of the 70 people who ate the potato salad at a school picnic, 63 developed symptoms of food poisoning. Which of the following best describes the attack rate? a. 63% b. 70% c. 90% d. 100%

c. 90% The attack rate is the proportion of persons exposed to an agent who develop the disease. Because 63 of the 70 persons became ill, the attack rate is 63:70, or 90%. Sixty-three (63) is the number of clients who developed symptoms of food poisoning. Seventy (70) is the total number of people who ate the potato salad at the school picnic. These numbers need to be used as part of the formula to calculate the attack rate. One hundred (100) does not represent any part of the formula that would be used to calculate the attack rate.

A woman is sitting in a corner of the clinical waiting room, crying audibly. The nurse asks, "What's wrong? Can I help?" The woman responds, "They just told me I have a positive mammogram and I need to see my doctor for follow-up tests. I know I'm going to die of cancer. How can I tell my family?" Which of the following information does the nurse need to know in order to help the woman cope with this finding? a. The negative predictive value of mammography b. The positive predictive value of mammography c. The reliability of mammography d. The validity of mammography

b. The positive predictive value of mammography The positive predictive value is the proportion of persons with a positive test who actually have the disease, interpreted as the probability that an individual with a positive test has the disease. A negative predictive value is the proportion of persons with a negative test who are actually disease free. Reliability relates to the consistency or repeatability of the measure. Validity in a screening test is typically measured by sensitivity (how accurately it identifies those with the condition) and specificity (how accurately the test identifies those without the condition).

Which of the following best describes the purpose of local health departments making unannounced inspections of local restaurants? a. To enforce local laws and regulations. b. To ensure compliance. c. To provide oversight to potential exposure. d. To monitor employee safety.

b. To ensure compliance. Ensuring compliance refers to the process of making certain that permitting requirements are met. Although this activity may be seen as a type of monitoring, the question asks for the purpose, which is to ensure compliance. Enforcement involves penalties such as fines or facility closure.

A nurse who works at the local hospital asks a public health nurse what might be causing her hands to become very sore and sensitive. Which of the following statements would be the most appropriate response by the public health nurse? a. "Ask the staff in employee health if there have been any other complaints." b. "Call the local health department about any recent problems at the hospital." c. "Check the material safety data sheet on any chemicals you have used recently." d. "See the nurse's health care provider for a complete work-up."

c. "Check the material safety data sheet on any chemicals you have used recently." Employees have the right to know about hazardous chemicals with which they work. Employers must maintain a list of all hazardous chemicals used on the premises, along with a material safety data sheet that addresses health risks related to these chemicals, as well as information about safe use and handling. After checking the material safety data sheets, the next steps would be to connect with a health care provider which could include staff in employee health, a local health department, or one's own health care provider. The first step should be to determine what exposure occurred.

A man loudly protests his increased property tax bill right after the public health department has made a plea for more funds. "Why," he asks, "should my tax dollars be used to pay for their children to be immunized?" Which of the following would be the best response by the nurse? a. "Immunizations are required by law, and if their parents can't afford it, you and I will have to pay for it." b. "It's just the right thing to do." c. "Only by making sure most kids are immunized can we stop epidemics that might hurt all of us." d. "We're a religious God-fearing community, and we take care of each other."

c. "Only by making sure most kids are immunized can we stop epidemics that might hurt all of us." Herd immunity is the resistance of a group of people to invasion and spread of an infectious agent because a high proportion of individual members of a group are resistant to the infection. Higher immunization coverage will lead to greater herd immunity, which in turn will block the further spread of the disease. The purpose of immunization laws is to promote herd immunity. Receiving immunizations helps to block the further spread of the disease as more members of the community become resistant to the infection.

A client is crying softly and saying, "What did I do to deserve this punishment, Lord?" Which of the following responses by the nurse would be the most appropriate? a. "God doesn't punish people. You're sick just because of bad luck." b. "I can call the hospital chaplain to help you talk about these feelings." c. "What can I do to be helpful to you right now?" d. "Would you like to confess your sins and repent so this illness will go away?"

c. "What can I do to be helpful to you right now?" Some clients may view their illness as punishment for misdeeds and may have difficulty accepting care from nurses who do not share their beliefs. Because the nurse may not be a member of the client's religious faith group, an open-ended response showing caring is the most appropriate statement. The most therapeutic response from the nurse is an open-ended question. This allows the client to share information and not feel like his or her actions are being judged by the nurse. Also, this allows the nurse to not give advice or offer false information to the client.

A 40-year-old Bosnian, Muslim woman who does not speak English presents to a community health center in obvious pain. She requests a female health care provider. Through physical gestures, the woman indicates that the pain is originating in either the pelvic or genital region. Which of the following interpreters would be the most appropriate in this situation? a. A Bosnian male who is certified as a medical interpreter. b. A female from the client's community c. A female who does not know the client. d. The client's 20-year-old daughter

c. A female who does not know the client. Although having experience in medical interpretation is important, in many cultures it is inappropriate to have a male interpreter for females. This client has specifically requested a female provider; therefore, one might anticipate that the client will not be as forthcoming with a male interpreter. Regardless of certification and ability, the interpreter cannot interpret information the client may withhold because she feels it inappropriate to discuss private matters in front of a male. This client may also feel it inappropriate to have private matters interpreted by her daughter (especially if they are of a sexual nature or if they involve infidelity). Additionally, to avoid a breach of confidentiality, the nurse should avoid using an interpreter from the same community as the client.

Which of the following types of study should the nurse researcher choose if the goal is to identify the long-term benefits and risks of a particular nursing intervention for senior citizens living in the community? a. Cross-sectional study b. Ecological study c. Clinical trial d. Retrospective analysis

c. Clinical trial The goal of a clinical trial is to evaluate the effectiveness of an intervention. Clinical trials are generally the best way to show causality. A cross-sectional study provides a snapshot of a population or group. An ecological study is a bridge between descriptive and analytic epidemiology, looking at variations in disease rates by person, place, or time and trying to determine if there is a relation of disease rates to variations in rates for possible risk factors. A retrospective analysis relies on existing records to define a cohort that has been exposed or unexposed at some point in the past.

A nurse is administering a tuberculosis (TB) skin test to a client who has acquired immune deficiency syndrome (AIDS). Which of the following results should the nurse anticipate when using this screening test? a. Decreased positive predictive value. b. Decreased reliability c. Decreased sensitivity d. Decreased specificity

c. Decreased sensitivity Persons with immune deficiencies may have a negative tuberculosis skin test even though they are infected. Sensitivity is the extent to which a test identifies those individuals who have the condition being examined. AIDS is an acquired immune deficiency; thus, clients with AIDS may have a false-negative response to TB skin tests; that is, they have the disease but the test is not sensitive enough to detect infection in these individuals. Therefore, there is decreased sensitivity with those clients. A positive predictive value is the proportion of persons with a positive test who actually have the disease, interpreted as the probability that an individual with a positive test has the disease. In this case, it is likely that if the individual has the disease, it will not be detected. Reliability relates to the consistency or repeatability of the measure. Because of the client's altered immune status, it is likely that the same results would be obtained if the test was to be repeated. Specificity indicates how accurately the test identifies those without the condition or trait. In this case, it is likely that a false-negative reading would be obtained, so the specificity may not be accurate.

Which of the following actions would a nurse take to reduce the high incidence of coronary artery disease (CAD) in a community? a. Introduction of a heart-healthy curriculum beginning in the first grade, presentations on diet and exercise for the community at large, and special education sessions for high-risk populations b. Provision of online activities related to prevention of cardiac disease, smoking reduction programs, and blood pressure screenings. c. Distribution of handouts, including age-appropriate games, self-assessments, and education on heart-healthy lifestyles; availability of community screenings for hyperlipidemia in persons age 35 and older; and walking programs for those affected with CAD. d. Enrollment of clients with CAD into cardiac rehabilitation programs, routine evaluation of effectiveness of CAD treatment regimens, and participation in clinical trials that evaluate interventions for those diagnosed with CAD.

c. Distribution of handouts, including age-appropriate games, self-assessments, and education on heart-healthy lifestyles; availability of community screenings for hyperlipidemia in persons age 35 and older; and walking programs for those affected with CAD. Distributing handouts includes all three levels of prevention to target all members of the population. Targeting all members of the population and implementing all levels of prevention increase the likelihood of positive outcomes for the community as a whole. Education in schools, the community, and high-risk populations focuses only on primary prevention activities. Online activities focus only on primary and secondary preventions. Efforts focused only on those who already have CAD are not primary prevention.

A nurse reports that in comparison to all the children in a particular school, the children who are members of the Cub Scouts have 0.3 risk for obesity before entering the sixth grade. Which of the following recommendations would the nurse make to the new parents of two boys who had just moved into this school's neighborhood? a. Discourage the parents from enrolling their sons in Cub Scouts because of the risk. b. Don't say anything about Cub Scouts, because it isn't relevant to nursing care. c. Encourage the parents to enroll their sons in Cub Scouts. d. Share the finding and let the parents draw whatever conclusions they feel appropriate.

c. Encourage the parents to enroll their sons in Cub Scouts. Relative risk is an estimation of the risk of acquiring a problem for those who are exposed compared with those who are unexposed. As the risk for obesity is less for those that are members of Cub Scouts, joining the group is protective and reduces the incidence among members. Parents should be encouraged to enroll their sons in Cub Scouts as joining the group is protective and reduces the risk for becoming obese. The finding should be interpreted for the parents, and the protective aspect of joining the group should be explained.

A nurse is investigating a bacterial illness that has caused a health problem in the community. Only some of the people exposed to the bacteria have become ill. Which of the following factors best explains why this would have happened? a. Chemical agent factors b. Environmental factors c. Host factors d. Physical agent factors

c. Host factors The epidemiologic triangle includes the agent, host, and environment. Only differences in host factors can explain why some became ill and some were able to fight off the bacterial infestation. The epidemiologic triangle includes the agent, host, and environment. The bacteria were the agent, so chemical and physical agents are not relevant. The environment was apparently the same for everyone, since all were exposed to the bacteria.

A nurse practitioner has established a mobile clinic to vaccinate adults against influenza. Which of the following phases of the nursing process is being implemented? a. Assessment b. Planning c. Intervention d. Evaluation

c. Intervention Giving immunizations is an intervention that will increase the number of people who will be vaccinated. Intervention includes coordinating medical, nursing, and public health actions to meet the client's needs. Assessment would have been done earlier in the process because this was needed to determine that a problem existed and that interventions were needed. Planning occurs when the nurse looks at community policy and laws as methods to facilitate the care needs for the client. When criteria that includes the immediate and long-term responses of the client as well as the recidivism of the problem for the client are examined, it is part of the evaluation phase.

A nurse executive is implementing evidence-based practice at a community nursing center that serves a large Cuban immigrant population. Which of the following actions would be the most helpful for the nurse? a. Review clinical policies with cultural competency experts from the local university. b. Conduct weekly staff meetings to discuss which methods work best when helping immigrants. c. Invite Cuban immigrants from the community to serve on the center's advisory board. d. Send personnel to conferences and seminars that focus on treating Hispanic immigrants.

c. Invite Cuban immigrants from the community to serve on the center's advisory board. To determine whether practice is serving the needs of the population, the nurse will need to consult the population served. This means that evidence should be applied with input from the community. For example, decisions related to the services to be offered in a nurse-managed clinic should be made with input from the clinic's advisory board, which should include community leaders and consumers of the clinic's services. Information about the population needs to come from the population itself, not from cultural competency experts within the community or from the staff members. Cuban immigrant needs may not be the same as those of Hispanic immigrants from other places such as Mexico or Spain.

Which of the following statements describes how nursing in the community is more challenging than nursing in an acute care setting? a. There is limited access to information useful to the nurse in giving care in the community. b. More paperwork and forms are required when giving care in the home. c. It is more challenging to control the environment in the community. d. Specialization isn't possible in the community setting.

c. It is more challenging to control the environment in the community. In the community, nurses often use epidemiology, since the factors that affect the individual, family, and population group cannot be as easily controlled as in acute care settings. It is essentially impossible to control the environment in the community. Nurses working in the community have multiple resources that can be used to access information about the community. There may be additional paperwork and forms to complete in the home environment, but the lack of control of the environment is a larger challenge than the completion of paperwork. Community health nursing is recognized as a specialty within nursing.

A nurse gives detailed information on how to apply for Medicaid to a new mother who moved to the United States from Russia about 10 years ago. The nurse's next client is an African-American mother of newborn twins who worked until the children were born. The nurse knows the woman is eligible to maintain her insurance after her employment was lost and does not discuss insurance options at all. Which of the following errors is being made by the nurse? a. Covert intentional prejudice b. Covert unintentional prejudice c. Overt intentional prejudice d. Overt unintentional prejudice

c. Overt intentional prejudice The nurse may have assumed that the African-American mother knew the available resources and could negotiate for assistance on her own and that the immigrant Russian woman had no experience negotiating government programs and thus needed the nurse to advocate for her and inform her of the programs available to her. The nurse, not knowing the health-seeking behaviors of either client, stereotyped both women and intentionally used her informational power to help one client while denying assistance to the other client. Covert prejudice involves subtle or passive acts of prejudice. This can occur intentionally or unintentionally. In this case, the actions of the nurse were intentional, which means it was an overt behavior. Overt prejudice includes any action that intends to give unequal treatment to an individual or group. Given the scenario, it appears that the action of the nurse was intentional, not unintentional.

This year 600 of 8000 young women age 17 to 20 years at a university health center tested positive for a sexually transmitted infection (STI). Which of the following terms best describes this data? a. An epidemic b. Incidence c. Prevalence d. Risk

c. Prevalence Prevalence is the measure of existing disease in a population at a particular time. Because we do not have baseline data, we have no way to conclude that this is an epidemic with higher-than-normal results from the screening. Incidence refers to new cases, whereas prevalence means all cases. We don't know whether the finding represents the first time a woman was told she had an STI or whether she had previously been diagnosed with the problem. Therefore, we can't say whether these are new cases (incidence), but the results do represent all cases (prevalence). Risk is the probability of developing an STI, but no risk factors are discussed.

A nursing administrator wants to develop a work environment conducive to the implementation of evidence-based practice (EBP). Which of the following actions would best achieve this goal? a. Conducting market research to determine customer satisfaction with EBP. b. Eliciting opinions from nurses on how EBP will affect workload. c. Purchasing computers and Internet access for use by employees. d. Sending staff to conferences related to incorporation of EBP into practice

c. Purchasing computers and Internet access for use by employees A lack of computers and Internet access can create a barrier to implementation of EBP in community-based nursing agencies. If these are provided, nurses can quickly access current evidence-based findings and recommendations. Conducting market research will not change the work environment to make it more conductive to implementing EBP. Learning about nurses' opinions about how it will affect workload would not impact the work environment and make it more conducive to implementing EBP. Rather the administrator would need to discuss the benefits of the use of EBP with the staff. Sending staff to a conference, although it may change staff attitudes, would not necessarily change the work environment.

A nurse wants to help keep the community environment safe. Which of the following is the most appropriate action for the nurse to take? a. Conserve water by bathing less often. b. Downsize to a smaller living space. c. Reduce, reuse, and recycle. d. Replace all his or her appliances with new energy-efficient ones.

c. Reduce, reuse, and recycle. All persons can engage in minimizing their impact on the environment by reducing, reusing, and recycling the individual products they currently consume. Although it would be nice to replace all appliances for energy-efficient newer models, most people can't afford that—furthermore, it is an environmental issue to discard the used models. Similarly, downsizing to a smaller living space could help the environment, because the nurse would use less energy, but again, not everyone can afford all the expenses involved in moving.

A public health nurse is assisting the community in dealing with the effects of lead paint poisoning. Which of the following activities would the nurse most likely complete? (Select all that apply.) a. Administering medications to those with signs of lead poisoning b. Assessing community members for any health problems. c. Setting up a blood screening program with the local health department. d. Encouraging local landlords to improve the condition of their housing. e. Educating the public on the dangers of lead paint.

c. Setting up a blood screening program with the local health department. d. Encouraging local landlords to improve the condition of their housing. e. Educating the public on the dangers of lead paint. The nurse's role is to understand the roles of each respective agency and organization, know the public health laws, and work with the community to coordinate services to address the community's needs. Other needed interventions include organizing a blood-lead screening program through the local health department, educating local health providers to encourage them to systematically test children for lead poisoning, and working with local landlords to improve the condition of their housing stock. Educating the community of the hazards of lead paint falls within the roles of the public health nurse. Administering medications to those with signs of lead poisoning is not a typical role of the nurse when dealing with lead poisoning. Assessing community members for any health problems is too broad of an intervention and does not address the specific need of the nurse working with lead poisoning in the community.

An occupational health nurse at a local factory is using primary prevention strategies to reduce the environmental health risks among the employees. Which of the following activities would the nurse most likely implement? a. Checking radiation detectors to monitor for unsafe levels of radiation exposure. b. Irrigating the eyes of an employee who has had a chemical splash to the face. c. Teaching new employees who will work outdoors about the signs and symptoms of heat-related illness. d. Using spirometry to rule out obstructive or restrictive lung disease for workers who will be wearing mask respirators.

c. Teaching new employees who will work outdoors about the signs and symptoms of heat-related illness. Education is a primary preventive strategy. When examining the sources of environmental health risks in communities and planning intervention strategies, it is important to apply the basic principles of disease prevention. Checking radiation detectors and using spirometry are both examples of screening for potential exposure which is part of secondary prevention. Irrigating the eyes of an employee who had a chemical splash to the face addresses tertiary prevention as a problem has already occurred.

A nurse advises a client with osteoporosis to have three servings of milk or dairy products daily. Which of the following levels of prevention is being used by the nurse? a. Primary prevention b. Secondary prevention c. Tertiary prevention d. Treatment, but not prevention

c. Tertiary prevention Interventions that prevent worsening of a condition are tertiary prevention activities. In this instance, the client already has a health problem (osteoporosis). By advising adequate dairy intake, the nurse aims to ensure that enough calcium is available to limit worsening of the osteoporosis. Primary prevention would focus on interventions that reduce the risk of one getting the disease. Secondary prevention addresses screening tests to detect the disease at the earliest stage possible. Treatment is not a level of prevention.

A nursing staff has successfully screened for diabetes in the community. Which of the following might best persuade the health board to increase funding for diabetic clinics in this community? a. An epidemic of diabetes is now recognized and must be addressed. b. The incidence of diabetes is now higher than previously recognized in the community. c. The prevalence of diabetes is now higher than previously recognized in the community. d. The risk for diabetes in the community could decrease if funding is received.

c. The prevalence of diabetes is now higher than previously recognized in the community. Prevalence is a fairly stable number. Prevalence proportions reflect duration in addition to the risk for getting the disease. Prevalence is useful in planning health care services because it is an indication of the level of disease existing in the population and therefore the size of the population in need of services. An epidemic occurs when the rate of disease, injury, or other conditions exceeds the usual level of that condition. Incidence is affected only by factors related to the risk for developing the disease and not survival or cure. The nurse is proposing to increase funding for diabetic clinics, and having more diabetic clinics would assist those who already have the disease, so it would have limited impact on decreasing the risk of developing the disease among members of the community.

Persons in an auditorium may have been exposed to a disease. If they are infected, it is crucial that they receive immediate treatment and not take the disease home to their families. Which of the following characteristics would be most important to consider when selecting the screening test to be used? a. The negative predictive value b. The positive predictive value c. The sensitivity of the test d. The specificity of the test

c. The sensitivity of the test Because it is most important to identify every case, the sensitivity of the test is crucial. High sensitivity is needed when early treatment is important and when the identification of every case is important. A negative predictive value is the proportion of persons with a negative test who are actually disease free. A positive predictive value is the proportion of persons with a positive test who actually have the disease, interpreted as the probability that an individual with a positive test has the disease. Specificity indicates how accurately the test identifies those without the condition or trait.

A school nurse wants to decrease the incidence of obesity in elementary school children. Which of the following describes a secondary prevention intervention that the nurse could implement?

c. Weighing students to identify those who are overweight. Secondary prevention refers to interventions that increase the probability that a person with a condition will have the condition diagnosed early. Health screenings are the mainstay of secondary prevention. Weighing students and assessing whether the weight is higher than recommended will allow for early intervention so that obesity may be avoided. Giving a presentation and designing a game are both examples of primary prevention. Placing overweight students on a diet is an example of tertiary prevention.

The nurse has been reading extensively on a particular clinical problem, using both the closest medical library and the Internet. Which of the following would be the most helpful source? a. A journal with a whole issue devoted to research on that clinical problem. b. A randomized controlled clinical trial related to that clinical problem. c. A researcher who has built a career on studying that clinical problem. d. A systematic review related to the clinical problem.

d. A systematic review related to the clinical problem. A systematic review is an approach to identifying, appraising, and synthesizing research evidence to evaluate and interpret all available research that is relevant to a particular research question. Systematic reviews can be accessed from most databases. Systematic reviews require more rigor and contain less opinion of the author than typical reviews of the literature. An entire journal devoted to research on a clinical problem may be helpful, but it is unknown what type of research is being published in that journal. A well-designed systematic review can provide stronger evidence than a single randomized controlled trial. A researcher alone does not provide the best evidence, rather one would need to look at the information that has been published by the researcher in peer-reviewed journals.

Several small communities have applied for grant funding from the state department of health to help decrease their teenage pregnancy rate. Which of the following communities should the nurse suggest receive funding first? a. Community A—with 23 single teenage pregnancies in a city of 500 b. Community B—with 45 single teenage pregnancies in a city of 1000 c. Community C—with 90 single teenage pregnancies in a city of 2000 d. Community D—with 90 single teenage pregnancies in a city of 1500

d. Community D—with 90 single teenage pregnancies in a city of 1500 The pregnancy rates of A, B, and C are 45-46:1000, whereas the rate in Community D is 60:1000. Without doing any actual math, it should be fairly obvious that 23:500, 45:1000, and 90:2000 are all about the same proportion but that 90:1500 is a larger proportion.

A male nurse had a habit of sitting with the lower part of one leg resting over the knee of his opposite leg when collecting a client's history. He stopped doing this around specific clients after being told that they were offended when he exposed the sole of his foot (shoe) to their face. Which of the following was exhibited by the nurse when he changed his behavior? a. Cultural accommodation b. Cultural imposition c. Cultural re-patterning d. Cultural skill

d. Cultural skill Cultural skill is the effective integration of cultural knowledge and awareness to meet client needs—in this case, the clients need to not be offended by having the bottom of the nurse's foot or shoe in view of the client's face. The nurse using cultural skill makes sure that nonverbal communication techniques take into consideration the client's use of body language and space. Cultural accommodation involves negotiation with clients to include aspects of their folk practices with the traditional health care system to implement essential treatment plans. Cultural imposition is the process of imposing one's values on others. Cultural re-patterning is working with clients to make changes in their health practices if cultural behaviors are harmful or decrease their well-being.

The nurse is examining blood lead levels in school-age children 1 year after a community-wide education intervention. Which of the following phases of the nursing process is being implemented? a. Assessment b. Diagnosis c. Intervention d. Evaluation

d. Evaluation In this instance, the nurse is evaluating the results of the intervention to determine whether goals were reached. Assessment would have been done earlier in the process because this was needed to determine that a problem existed and that interventions were needed. Diagnosis occurs when the disease and environmental factors are related to the diagnosis. When intervention is used, the nurse coordinates medical, nursing, and public health actions to meet the client's needs.

A Spanish-speaking family comes to the public health department. No one in the family speaks English, and nobody at the health department speaks Spanish. Which of the following actions should be taken by the nurse? a. Attempt communication using an English-Spanish phrase book. b. Call the local hospital and arrange a referral. c. Emphatically state, "No hablo Español" (I don't speak Spanish). d. Obtain an interpreter to translate.

d. Obtain an interpreter to translate. Communication with the client or family is required for a careful assessment. When nurses do not speak or understand the client's language, they should obtain an interpreter. The nurse must use strategies that will allow effective communication with the client. The client has the right to receive effective care, to judge whether the care was appropriate, and to follow up with appropriate action if the expected care was not received. The nurse must contact an interpreter in order to provide the best care for the client—attempting communication using a book, stating that he or she does not speak English, and arranging for a referral do not address the priority action of finding an interpreter.

A high school student considering a job in the restaurant industry after graduation asks a nurse about workplace safety issues. Which of the following acts would the nurse most likely discuss with the student? a. Chemical Safety Information, Site Security, and Fuels Regulatory Act b. Comprehensive Environmental Response, Compensation, and Liability Act c. Food Quality Protection Act d. Occupational Safety and Health Act

d. Occupational Safety and Health Act The Occupational Safety and Health Act (OSHA) was passed to ensure worker and workplace safety in all employment settings, including restaurants. The Chemical Safety Information, Site Security, and Fuels Regulatory Act, Comprehensive Environmental Response, Compensation, and Liability Act, and Food Quality Protection Act may contribute to safety but do not have the full oversight of OSHA.

During an outbreak of hepatitis A, nurses are giving injections of hepatitis A immunoglobulin to selected susceptible persons. Which of the following best describes the type of immunity that will follow the administration of these injections? a. Active immunity b. Acquired immunity. c. Natural immunity d. Passive immunity

d. Passive immunity Passive immunity refers to immunization through the transfer of a specific antibody from an immunized individual to a non-immunized individual, such as the transfer of antibody by the administration of an antibody-containing preparation (immune globulin or antiserum). Passive immunity from immune globulin is almost immediate but short-lived. It often is induced as a stopgap measure until active immunity has had time to develop after vaccination. Active immunity occurs as antibodies develop due to exposure to the antigen. Acquired immunity is the resistance acquired by a host as a result of previous natural exposure to an infectious agent. Natural immunity refers to a species-determined, innate resistance to an infectious agent.

A principal comments to the school nurse that it seems there are a lot more problems with asthma among the students than there were before the school was remodeled a couple of years ago. The nurse investigates the principal's observation by reviewing all the school records to determine visits to the health office because of asthma by week and month for the past 5 years. Which of the following best describes the type of study the nurse is conducting? a. Descriptive epidemiological study b. Ecological study c. Prospective cohort study d. Retrospective cohort study

d. Retrospective cohort study Retrospective cohort studies rely on existing records to define a cohort that is classified as having been exposed or unexposed at some time in the past. In this case, the issue is whether there is some health risk in the new building addition that is increasing frequency of visits to the school nurse because of asthma. In a descriptive epidemiological study, the disease is investigated in terms of person, place, and time. An ecological study is a bridge between descriptive and analytic epidemiology, looking at variations in disease rates by person, place, or time and trying to determine if there is a relation of disease rates to variations in rates for possible risk factors. A prospective cohort study, or subjects who do not have the outcome under investigation, are classified on the basis of the exposure of interest at the beginning of the follow-up period.

A health care worker tells a nurse, "It does no good to try to teach those Medicaid clients about nutrition because they will just eat what they want to no matter how much we teach them." Which of the following is being demonstrated by this statement? a. Cultural imposition b. Ethnocentrism c. Racism d. Stereotyping

d. Stereotyping Stereotyping occurs when someone attributes certain beliefs and behaviors about a group to an individual without giving adequate attention to individual differences. In this instance, the health care worker makes the assumption that clients with low incomes are not educable. The health care worker is guilty of making another assumption as well: noncompliance among other Medicaid clients the worker has known may have been related to an inability to afford nutritious food. Cultural imposition is the belief in one's own superiority, or ethnocentrism, and is the act of imposing one's values on others. Ethnocentrism is a type of cultural prejudice at the population level which involves the belief that one's own group determines the standards for behavior by which all other groups should be judged. Racism refers to the belief that persons who are born into a particular group are inferior in intelligence, morals, beauty, or self-worth.

A nurse manager wants to facilitate incorporation of evidence-based practice (EBP) in the clinical setting. Which of the following would be the best strategy to accomplish this goal? a. Eliminate all protocols and standards that are not evidence based. b. Encourage group reflection on the ideals and expectations of nursing care. c. Refer agency nurses to Internet sources of research findings. d. Support nurses using practice-oriented research findings in decision-making.

d. Support nurses using practice-oriented research findings in decision-making. EBP demands changes. It requires incorporating more practice-oriented research and more collaboration between clinicians and researchers. Emphasis should be on decision- making using the varied sources of evidence. The environment and climate must be supportive in order to implement EBP. Rather than eliminating protocols and standards that are not evidence based, the nurse manager should make it a priority to begin to update these practices based on EBP. Self-reflection on one's own nursing practice and how EBP can be implemented would be more important than group reflection on the large ideals of nursing practice. Referring the nurses to the Internet for ideas is helpful only if evidence-based practice sites are accessed, and most Internet sites are not EBP sites.

Between 2010 and 2015, 1000 of 10,000 young women age 17 to 20 years at a university tested positive for a sexually transmitted infection (STI). Of the 1000 diagnosed STIs, 300 were gonorrhea and 500 were chlamydia. Which of the following statements best summarizes these findings? a. The proportion of cases of gonorrhea to all STIs was 300:1300. b. The proportion of cases of gonorrhea to chlamydia was 300:500. c. The proportion of cases of gonorrhea to all STIs was 50%. d. The proportion of STIs to the total population was 100:1000.

d. The proportion of STIs to the total population was 100:1000. A proportion is a ratio in which the denominator includes the numerator. If the proportion is small, we can express the number per 1000. The answer of 100:1000 correctly summarizes that 1000 of 10,000 (or 100:1000) young women had the problem. The answer of 300:1300 adds the total of the numerator to the denominator, which is unnecessary because the gonorrhea cases were already included in the denominator. In the answer of 300:500, the ratio comparing gonorrhea to chlamydia does not meet the epidemiological definition of proportion (i.e., the denominator must contain the numerator). Although proportions may be expressed as percentages, in the answer of 50%, the percentage reflects the number of gonorrhea cases to all STIs, which doesn't summarize the total STI problem.

A teacher recommends that surveys to obtain data on drug use be given to high school students when they meet for various school organizations. Which of the following best describes why the nurse would reject this suggestion? a. This method of data collection would result in classification bias. b. This method of data collection would result in confounding bias. c. This method of data collection would result in personal bias. d. This method of data collection would result in selection bias.

d. This method of data collection would result in selection bias. Any study is subject to bias resulting from selective choice. There may be a difference between students who choose to belong to an organization and students who choose not to join an organization. Selection bias occurs when selection procedures are not representative of the population as a whole. In this instance, the goal is to determine drug use of all students at the school. If only students who join school organizations are selected, those who do not join organizations will not be represented. Classification bias occurs once subjects are enrolled in the study and relates to how information is collected. Confounding bias results from the relationship between the outcome and the study factor and some third factor that was not accounted for. Personal bias deals with personal opinions and attitudes of the researcher which may interfere with the proper selection of the participants.

A nurse wants to have a better understanding of the physiological effects of selected chemicals. Which of the following individuals would provide the most useful information to the nurse? a. Chemist b. Epidemiologist c. Pharmacist d. Toxicologist

d. Toxicologist Toxicology is the basic science that studies the health effects associated with chemical exposures. A chemist, epidemiologist, or pharmacist would probably be able to provide some information, but the toxicologist would most likely provide information related to physiological processes.

A nurse has only a regular blood pressure cuff when conducting a health screening for all of the residents of a community. Which of the following may be lacking when obtaining blood pressure readings? a. Reliability b. Sensitivity c. Specificity d. Validity

d. Validity Validity is the accuracy of a test or measurement, or how closely it measures what it claims to measure. With only one regular BP cuff, the nurse cannot obtain accurate measurements on those who are extremely obese or extremely thin. A thigh cuff and a pediatric cuff would allow the nurse to obtain accurate—that is, valid—measurements. Reliability refers to the consistency or repeatability of the measure. If the wrong size blood pressure cuff is used, it is likely that the same wrong blood pressure would be repeatedly obtained. Sensitivity and specificity are the two ways that validity can be described. Sensitivity quantifies how accurately the test identifies those with the condition or trait. Specificity indicates how accurately the test identifies those without the condition or trait. With the wrong size blood pressure cuff, the sensitivity and specificity of the screening would be in question.

A nurse is examining the various factors that lead to disease and suggesting several areas where nurses could intervene to reduce future incidence of disease. Which of the following models would the nurse most likely use? a. Epidemiologic triangle b. Health promotion c. Levels of prevention d. Web of causality

d. Web of causality The web of causality model recognizes the complex interrelationships of many factors interacting to increase or decrease the risk for disease. Causal relationships (one thing or event causing another) are often more complex than the epidemiologic triangle conveys. Levels of prevention are actions taken to improve health outcomes. Health promotion addresses health improvement not identification of where nurses could intervene to reduce future incidence of disease.


Set pelajaran terkait

IB Biology Unit 7: Carbohydrates and Lipids

View Set

ch. 13 Outcome identification and planning

View Set

Periódusos rendszer - elemek neve, vegyjele, rendszáma

View Set

CH 40 Nursing Care of the Child with an Alteration in Gas Exchange/Respiratory Disorder

View Set

Aud16.17.18.21 - 1.2.5.6.7.8.9.10.11.12.13.14.15

View Set

Mansa Musa (the World's Richest Man)

View Set

9th Grade Biology 1.1 What is Biology

View Set